中二數 我真係唔識

2007-07-31 8:42 am
以最簡形式寫出下列各比

1.)1.7kg : 680mg

2.)1 m(2次) :1 cm(2次)

3.) 1 km/h : 1 m/s

求未知數

4.)2/3 : 3/4 = 1:y

5.)電線長30cm 以 4 : 5 : 6 分成3部分 求每部分的長度。

6.) 某校有學生1500人 男生同女生之比係4 : 6。已知5/6的女生及4/5 的男生有配戴眼鏡。

a.) 求男生及女生的人數。

b.) 求配戴眼鏡的男生及女生人數 。


唔該幫幫手呀......我數學好差 /_\\

回答 (2)

2007-07-31 9:32 am
✔ 最佳答案
求比例最緊要留意單位.
1. 1.7kg : 680mg
= 1700 g : 0.680 g ----------->[kg = 1000g ; mg = (1 / 1000) g = 0.001g]
= 2500 : 1

2. 1 m^2 : 1 cm^2 -------> (^2 = 2 次方)
= 1 x (100^2) m^2 : 1 cm^2
= 10000 : 1

3. 1 km/h : 1 m/s
= 1 x (1000/3600) m/s : 1 m/s -------------> (km x 1000 => m; h x 3600 => s)
= (1000/3600) : 1
= (5/18) : 1

4. <比例>另一個表達方式是用 <除法> :
例如: A : B = C : D , 可轉換成 A / B = C / D
所以問題解答如下:
2/3 : 3/4 = 1:y
(2/3) / (3/4) = 1/ y
(2/3) x (4/3) = 1 / y ------------> (除一個分數 = 乘上下調轉既分數)
8 / 9 = 1 / y
y = 9 / 8

5. 求每部分的長度:
設: x 段: y 段 : z 段 = 4 : 5 : 6
所以:
x 段長度 = 30 x [4/(4 + 5 + 6)] = 8 cm
y 段長度 = 30 x [5/(4 + 5 + 6)] = 10 cm
z 段長度 = 30 x [6/(4 + 5 + 6)] = 12 cm

6a. 求男生及女生的人數:
男生人數 = 1500 x [4/(4 + 6)] = 600 人
女生人數 = 1500 x [6/(4 + 6)] = 900 人

6b. 求配戴眼鏡的男生及女生人數
配戴眼鏡的男生人數 = 600 x (4/5) = 480 人
配戴眼鏡的女生人數 = 900 x (5/6) = 750 人
2007-07-31 9:48 am
1) 1 kg = 1 000 000 mg
=> 1.7kg = 1700000mg
so 1.7kg : 680mg = 1700000 mg : 680 mg
=> 2500 : 1

2.)1 m^2 :1 cm^2
=> 100cm x 100cm : 1cm x 1cm
=> 10000 cm^2 : 1 cm^2
=> 10000 : 1

3.) 1 km/h : 1 m/s
=> 1 km/ (60 mins x 60 sec) => 1000 m / 3600s => 10/36 m/s
=> 5/18 m/s : 1 m/s

4.)2/3 : 3/4 = 1:y
2/3 : 3/4 => 1 : 3/4 x 3/2 => 1 : 9/8
y=9/8

5.)電線長30cm 以 4 : 5 : 6 分成3部分 求每部分的長度。
電線長30cm 以 4 : 5 : 6 分成3部分=> (4 + 5 + 6) = 15份
= 每份 = 2cm
=> 4份 = 8cm
=> 5份 = 10cm
=> 6份 = 12cm

6.) 某校有學生1500人 男生同女生之比係4 : 6。已知5/6的女生及4/5 的男生有配戴眼鏡

a.) 求男生及女生的人數。
因為男生同女生之比係4 : 6
=> 男生佔 40% => 1500人 x 40% => 600人
=> 女生佔 60% => 1500人 x 60% => 900人

b.) 求配戴眼鏡的男生及女生人數 。
已知5/6的女生配戴眼鏡 = 900人 x 5/6 = 750人
4/5 的男生有配戴眼鏡 = 600人 x 4/5 = 480人

6c.) 求配戴眼鏡與不配戴眼鏡的學生人數之比
配戴眼鏡總人數 = 750 (女生) + 480(男生) = 1230人
不配戴眼鏡的學生人數 = 1500人 - 1230人 = 270人
配戴眼鏡與不配戴眼鏡的學生人數之比 = 1230 人: 270人


收錄日期: 2021-04-13 14:04:03
原文連結 [永久失效]:
https://hk.answers.yahoo.com/question/index?qid=20070731000051KK00238

檢視 Wayback Machine 備份